Difference between revisions of "2018 AIME I Problems/Problem 5"

(Solution 2)
(Solution 2)
Line 11: Line 11:
  
 
==Solution 2==
 
==Solution 2==
Do as done in Solution 1 to get <cmath>x^2+xy-2y^2=0</cmath> <cmath>\implies (\frac{x}{y})^2+\frac{x}{y}-2=0</cmath> <cmath>\implies \frac{x}{y}=\frac{-1\pm\sqrt{1+8}}{2}=1,-2</cmath> Do as done in Solution 1 to get <cmath>9x^2+6xy+y^2=3x^2+4xy+Ky^2</cmath> <cmath>\implies 6x^2+2xy+(1-K)y^2=0</cmath> <cmath>\implies 6(\frac{x}{y})^2+2\frac{x}{y}+(1-K)=0</cmath> <math></math>\implies \frac{x}{y}=<math> </math>\frac{-2\pm \sqrt{4-24(1-K)}}{12}<cmath> </cmath>\implies \frac{x}{y}=\frac{-2\pm 2\sqrt{6K-5}}{12}=\frac{-1\pm \sqrt{6K-5}}{6}<cmath>. If </cmath>\frac{x}{y}=1<cmath> then </cmath>1=\frac{-1\pm \sqrt{6K-5}}{6}<cmath> </cmath>\implies 6=-1\pm \sqrt{6K-5}<cmath> </cmath>\implies 7=\pm \sqrt{6K-5}<cmath> </cmath>\implies 49=6K-5<cmath> </cmath>\implies K=9<cmath>. If </cmath>\frac{x}{y}=-2<cmath> then </cmath>-2=\frac{-1\pm \sqrt{6K-5}}{6}<cmath> </cmath>\implies -12=-1\pm \sqrt{6K-5}<cmath> </cmath>\implies -11=\sqrt{6K-5}<cmath> </cmath>\implies 121=6K-5<cmath> </cmath>\implies 126=6K<cmath> </cmath>\implies K=21<math></math>. Hence our final answer is <math>21\cdot 9=\boxed{189}</math>
+
Do as done in Solution 1 to get <cmath>x^2+xy-2y^2=0</cmath> <cmath>\implies (\frac{x}{y})^2+\frac{x}{y}-2=0</cmath> <cmath>\implies \frac{x}{y}=\frac{-1\pm\sqrt{1+8}}{2}=1,-2</cmath> Do as done in Solution 1 to get <cmath>9x^2+6xy+y^2=3x^2+4xy+Ky^2</cmath> <cmath>\implies 6x^2+2xy+(1-K)y^2=0</cmath> <cmath>\implies 6(\frac{x}{y})^2+2\frac{x}{y}+(1-K)=0</cmath> <cmath>\implies \frac{x}{y}=\frac{-2\pm \sqrt{4-24(1-K)}}{12}</cmath> <cmath>\implies \frac{x}{y}=\frac{-2\pm 2\sqrt{6K-5}}{12}=\frac{-1\pm \sqrt{6K-5}}{6}</cmath>If <math>\frac{x}{y}=1</math> then <cmath>1=\frac{-1\pm \sqrt{6K-5}}{6}</cmath> <cmath>\implies 6=-1\pm \sqrt{6K-5}</cmath> <cmath>\implies 7=\pm \sqrt{6K-5}</cmath> <cmath>\implies 49=6K-5</cmath> <cmath>\implies K=9</cmath>If <math>\frac{x}{y}=-2</math> then <cmath>-2=\frac{-1\pm \sqrt{6K-5}}{6}</cmath> <cmath>\implies -12=-1\pm \sqrt{6K-5}</cmath> <cmath>\implies -11=\sqrt{6K-5}</cmath> <cmath>\implies 121=6K-5</cmath> <cmath>\implies 126=6K</cmath> <cmath>\implies K=21</cmath>Hence our final answer is <math>21\cdot 9=\boxed{189}</math>
 
-vsamc<math>\newline</math>
 
-vsamc<math>\newline</math>
 
-minor edit:einsteinstudent
 
-minor edit:einsteinstudent
 +
 
-style edit: yeaboi
 
-style edit: yeaboi
  

Revision as of 23:08, 24 August 2021

Problem 5

For each ordered pair of real numbers $(x,y)$ satisfying \[\log_2(2x+y) = \log_4(x^2+xy+7y^2)\]there is a real number $K$ such that \[\log_3(3x+y) = \log_9(3x^2+4xy+Ky^2).\]Find the product of all possible values of $K$.

Solution 1

Using the logarithmic property $\log_{a^n}b^n = \log_{a}b$, we note that \[(2x+y)^2 = 4x^2+4xy+y^2\] That gives \[x^2+xy-2y^2=0\] upon simplification and division by $3$. Factoring $x^2+xy-2y^2=0$ by Simon's Favorite Factoring Trick gives \[(x+2y)(x-y)=0\] Then, \[x=y \text{ or }x=-2y\] From the second equation, \[9x^2+6xy+y^2=3x^2+4xy+Ky^2\] If we take $x=y$, we see that $K=9$. If we take $x=-2y$, we see that $K=21$. The product is $\boxed{189}$.

-expiLnCalc

Solution 2

Do as done in Solution 1 to get \[x^2+xy-2y^2=0\] \[\implies (\frac{x}{y})^2+\frac{x}{y}-2=0\] \[\implies \frac{x}{y}=\frac{-1\pm\sqrt{1+8}}{2}=1,-2\] Do as done in Solution 1 to get \[9x^2+6xy+y^2=3x^2+4xy+Ky^2\] \[\implies 6x^2+2xy+(1-K)y^2=0\] \[\implies 6(\frac{x}{y})^2+2\frac{x}{y}+(1-K)=0\] \[\implies \frac{x}{y}=\frac{-2\pm \sqrt{4-24(1-K)}}{12}\] \[\implies \frac{x}{y}=\frac{-2\pm 2\sqrt{6K-5}}{12}=\frac{-1\pm \sqrt{6K-5}}{6}\]If $\frac{x}{y}=1$ then \[1=\frac{-1\pm \sqrt{6K-5}}{6}\] \[\implies 6=-1\pm \sqrt{6K-5}\] \[\implies 7=\pm \sqrt{6K-5}\] \[\implies 49=6K-5\] \[\implies K=9\]If $\frac{x}{y}=-2$ then \[-2=\frac{-1\pm \sqrt{6K-5}}{6}\] \[\implies -12=-1\pm \sqrt{6K-5}\] \[\implies -11=\sqrt{6K-5}\] \[\implies 121=6K-5\] \[\implies 126=6K\] \[\implies K=21\]Hence our final answer is $21\cdot 9=\boxed{189}$ -vsamc$\newline$ -minor edit:einsteinstudent

-style edit: yeaboi

Solution 3 (Official MAA)

Because $x^2+xy+7y^2=\left(x+\tfrac{y}{2}\right)^2+\tfrac{27}{4}y^2>0,$ the right side of the first equation is real. It follows that the left side of the equation is also real, so $2x+y>0$ and \[\log_2(2x+y)=\log_{2^2}(2x+y)^2=\log_4(4x^2+4xy+y^2).\] Thus $4x^2+4xy+y^2=x^2+xy+7y^2,$ which implies that $0=x^2+xy-2y^2=(x+2y)(x-y).$ Therefore either $x=-2y$ or $x=y,$ and because $2x+y>0,$ $x$ must be positive and $3x+y=x+(2x+y)>0.$ Similarly, \[\log_3(3x+y)=\log_{3^2}(3x+y)^2=\log_9(9x^2+6xy+y^2).\] If $x=-2y\ne 0,$ then $9x^2+6xy+y^2=36y^2-12y^2+y^2=25y^2=3x^2+4xy+Ky^2$ when $K=21.$ If $x=y\ne 0,$ then $9x^2+6xy+y^2=16y^2=3x^2+4xy+Ky^2$ when $K=9.$ The requested product is $21\cdot9=189.$

Video Solution

https://www.youtube.com/watch?v=iE8paW_ICxw

See Also

2018 AIME I (ProblemsAnswer KeyResources)
Preceded by
Problem 4
Followed by
Problem 6
1 2 3 4 5 6 7 8 9 10 11 12 13 14 15
All AIME Problems and Solutions

The problems on this page are copyrighted by the Mathematical Association of America's American Mathematics Competitions. AMC logo.png